2

JMO2022本選2

149
0
$$$$

 JMO2022-2を◯◯射的なノリの方針で解いたので解法を紹介します.

JMO2022 本選2

 関数$f:\mathbb{N}\rightarrow\mathbb{N}$であって,任意の$m,n\in \mathbb{N}$$f^{f(n)}(m)+mn=f(m)f(n)$をみたすものをすべて求めよ.

.
.
.
.
.
.
.
. (申し訳程度のネタバレ防止)
.
.
.
.
.
.
.

 $P(x,y)$で与式への代入を表す.

 $f$は単射である.

 正整数$a,b$$f(a)=f(b)$をみたすと仮定すると,$P(n,a)$$P(n,b)$を比較することで$a=b$が従う.

 $n$がすべての正整数を動くとき,$f(n)$$1$を除くすべての正整数値をとる.また,$f(1)=2$である.

 単射性から$f(n)$が最小値をとる唯一の$n$が存在するのでそれを$k$とおく.任意の$n\ne k$について$P(n,k)$$P(k,n)$から$f^{f(n)}(k)=f^{f(k)}(n)$が成り立ち,これと$f$の単射性から$f^{f(n)-f(k)}(k)=n$がわかる.したがって任意の$n\ne k$について$f(m)=n$なる正整数$m$が存在する.
 また,$f(c)=1$なる正整数$c$の存在を仮定すると,$P(n,c)$から$f(n)+c=f(n)$すなわち$c=0$が得られ矛盾.したがってこのような$c$は存在せず,よって$k=1$である.

 補題2から$P(n,1)$より$f(f(n))=2f(n)-n$が従う.$n$$1,2,\dots$を代入していくことにより帰納的に任意の正整数$n$について$\boxed{f(n)=n+1}$が成り立つことがわかる.十分性は明らか.


 以上です.嘘議論してそうで怖いですが多分回ってると信じてます.お読みくださりありがとうございました.

投稿日:29
更新日:29

この記事を高評価した人

高評価したユーザはいません

この記事に送られたバッジ

バッジはありません。

コメント

他の人のコメント

コメントはありません。
読み込み中...
読み込み中